Difference between revisions of "1997 AIME Problems/Problem 6"

m
(solution)
Line 1: Line 1:
 
== Problem ==
 
== Problem ==
Point <math>B</math> is in the exterior of the regular <math>n</math>-sided polygon <math>A_1A_2\cdots A_n</math>, and <math>A_1A_2B</math> is an equilateral triangle. What is the largest value of <math>n</math> for which <math>A_1</math>, <math>A_n</math>, and <math>B</math> are consecutive vertices of a regular polygon?
+
[[Point]] <math>B</math> is in the exterior of the [[regular polygon|regular]] <math>n</math>-sided polygon <math>A_1A_2\cdots A_n</math>, and <math>A_1A_2B</math> is an [[equilateral triangle]]. What is the largest value of <math>n</math> for which <math>A_1</math>, <math>A_n</math>, and <math>B</math> are consecutive vertices of a regular polygon?
  
 
== Solution ==
 
== Solution ==
{{solution}}
+
[[Image:1997_AIME-6.png]]
 +
 
 +
Let the other regular polygon have <math>m</math> sides. Using the interior angle of a regular polygon formula, we have <math>\angle A_2A_1A_n = \frac{(n-2)180}{n}</math>, <math>\angle A_nA_1B = \frac{(m-2)180}{m}</math>, and <math>\angle A_2A_1B = 60^{\circ}</math>. Since those three angles add up to <math>360^{\circ}</math>,
 +
 
 +
<cmath>\begin{eqnarray*}
 +
\frac{(n-2)180}{n} + \frac{(m-2)180}{m} &=& 300\\
 +
m(n-2)180 + n(m-2)180 &=& 300mn\\
 +
360mn - 360m - 360n &=& 300mn\\
 +
mn - 6m - 6n &=& 0
 +
\end{eqnarray*}</cmath>
 +
Using [[Simon's Favorite Factoring Trick|SFFT]],
 +
 
 +
<cmath>\begin{eqnarray*}
 +
(m-6)(n-6) &=& 36
 +
\end{eqnarray*}</cmath>
 +
Clearly <math>n</math> is maximized when <math>m = 7, n = \boxed{42}</math>.
  
 
== See also ==
 
== See also ==
 
{{AIME box|year=1997|num-b=5|num-a=7}}
 
{{AIME box|year=1997|num-b=5|num-a=7}}
 +
 +
[[Category:Intermediate Algebra Problems]]
 +
[[Category:Intermediate Geometry Problems]]

Revision as of 22:21, 21 November 2007

Problem

Point $B$ is in the exterior of the regular $n$-sided polygon $A_1A_2\cdots A_n$, and $A_1A_2B$ is an equilateral triangle. What is the largest value of $n$ for which $A_1$, $A_n$, and $B$ are consecutive vertices of a regular polygon?

Solution

1997 AIME-6.png

Let the other regular polygon have $m$ sides. Using the interior angle of a regular polygon formula, we have $\angle A_2A_1A_n = \frac{(n-2)180}{n}$, $\angle A_nA_1B = \frac{(m-2)180}{m}$, and $\angle A_2A_1B = 60^{\circ}$. Since those three angles add up to $360^{\circ}$,

\begin{eqnarray*} \frac{(n-2)180}{n} + \frac{(m-2)180}{m} &=& 300\\ m(n-2)180 + n(m-2)180 &=& 300mn\\ 360mn - 360m - 360n &=& 300mn\\ mn - 6m - 6n &=& 0 \end{eqnarray*} Using SFFT,

\begin{eqnarray*} (m-6)(n-6) &=& 36 \end{eqnarray*} Clearly $n$ is maximized when $m = 7, n = \boxed{42}$.

See also

1997 AIME (ProblemsAnswer KeyResources)
Preceded by
Problem 5
Followed by
Problem 7
1 2 3 4 5 6 7 8 9 10 11 12 13 14 15
All AIME Problems and Solutions